A soccer field (football pitch) has a length of 101.0 m and a
width of 70.2 m. Find the total area of the field in square
meters (m²) and convert this measurement to square yards
(yd2). Use the fact that 1 yard = 0.9144 m. Round your answer
to the nearest whole number.

Answers

Answer 1

Answer:

7754

Step-by-step explanation:

give brainiest, please. <3

Answer 2

Answer:

about 7,754 m

Step-by-step explanation:

101 * 70.2 = 7,090.2

7,090.2/0.9144 = 7,753.9


Related Questions

Use the property of real numbers to write the expression 2/5 x (-7)x 5/2

Answers

The simplified form of the expression 2/5 x (-7)x 5/2 would be -7.

How to solve the expression?

When two fractions are multiplying each other, the numerator of both fractions multiplies each other, likewise the denominator of both fractions respectively.

Use the property of real numbers to write the expression

[tex]\dfrac{2}{5} \times (-7) \times \dfrac{ 5}{2}\\\\\dfrac{10}{10} \times (-7) \\\\-7 \times 1\\\\= -7[/tex]

Therefore, The simplified form of the expression would be -7.

Learn more about an expression here:

https://brainly.com/question/1249625

#SPJ1

NO LINKS!! Please help me with this problem​

Answers

Answer:

23.1 years  (nearest tenth)

Step-by-step explanation:

Compound Interest Formula

[tex]\large \text{$ \sf A=P\left(1+\frac{r}{n}\right)^{nt} $}[/tex]

where:

A = final amountP = principal amountr = interest rate (in decimal form)n = number of times interest applied per time periodt = number of time periods elapsed

Given:

A = $1200P = $600r = 3% = 0.03n = 12 (as compounded monthly)t = years

Substitute the given values into the formula and solve for t:

[tex]\implies \sf 1200=600\left(1+\dfrac{0.03}{12}\right)^{12t}[/tex]

[tex]\implies \sf 1200=600\left(1.0025\right)^{12t}[/tex]

[tex]\implies \sf \dfrac{1200}{600}=\left(1.0025\right)^{12t}[/tex]

[tex]\implies \sf 2=\left(1.0025\right)^{12t}[/tex]

Take natural logs:

[tex]\implies \sf \ln 2=\ln \left(1.0025\right)^{12t}[/tex]

[tex]\implies \sf \ln 2=12t\ln \left(1.0025\right)[/tex]

[tex]\implies t=\dfrac{ \ln 2}{12 \ln (1.0025)}[/tex]

[tex]\implies t=23.13377513...[/tex]

[tex]\implies t=23.1\: \sf years \:\:(nearest\:tenth)[/tex]

The money will double in value in approximately [tex]\boxed{\sf 23.1}[/tex] years.

Used formula

[tex]\boxed{\sf A=P(1+\dfrac{r}{n})^{nt}}[/tex]

[tex]\\ \implies \sf 1200=600\left(1.0025\right)^{12t}[/tex]

[tex]\\ \implies \sf \dfrac{1200}{600}=\left(1.0025\right)^{12t}[/tex]

[tex]\\ \implies \sf 2=\left(1.0025\right)^{12t}[/tex]

Apply natural logarithm on both sides

[tex]\implies \sf \ln 2=\ln \left(1.0025\right)^{12t}[/tex]

[tex]\\ \implies \sf \ln 2=12t\ln \left(1.0025\right)[/tex]

[tex]\\ \implies t=\dfrac{ \ln 2}{12 \ln (1.0025)}[/tex]

[tex]\\ \implies t=23.1years[/tex]

A motorboat moves across a lake at a constant speed. When it begins, it is 64 km from the shore. After 17 minutes, it is 30 km from the shore. Which function describes the motorboat's distance from the shore?

A. y=-17x+30
B. y=2x+64
C. y=-17x+64
D. y=-2x+64

Please solve fast!
I am giving a lot of points!

Answers

Answer:

y = -2x + 64

Explanation:

The x represents the time in minutes and y represents distance in km.

Determine coordinates: (0, 64), (17, 30)

Find slope:

[tex]\sf slope : \dfrac{y_2 - y_1}{x_2- x_1} = \dfrac{\triangle y}{\triangle x} \ \ \ where \ (x_1 , \ y_1), ( x_2 , \ y_2) \ are \ points[/tex]

[tex]\rightarrow \sf slope \ (m) : \dfrac{30-64}{17-0} = -2[/tex]

Find equation:

[tex]\sf y- y_1 = m(x - x_1)[/tex]

[tex]\sf y - 64 = -2(x - 0)[/tex]

[tex]\sf y = -2x + 64[/tex]

The credit remaining on a phone card (in dollors) is a linear function of the total calling trime made with the card ( in minutes). The remaining credit after 22 minutes is $36.70, and the remaining credit after 52 mintues is $32.20. What is the remaining credit after 85 mintues of calls?

Answers

After 85 minutes of calls, there are $27.25 left on the card.

What is the remaining credit after 85 minutes of calls?

A linear equation in slope-intercept form is:

y = a*x + b

Where a is the slope.

If the line passes through two points (x₁, y₁) and (x₂, y₂) the slope is:

[tex]a = \frac{y_2 - y_1}{x_2 - x_1}[/tex]

In this case, we know that the line passes through the points (22, 36.7) and (52, 32,20)

(points of the form (time, dollars)).

So the slope is:

[tex]a = \frac{32.2 - 36.7}{52 -22} = -0.15[/tex]

The linear equation is then:

y = -0.15*x + b

To find the value of b, we use the point (22. 36.7)

36.7 = -0.15*22 + b

36.7 + 0.15*22 = b = 40

Then the linear equation is:

y = -0.15*x +40

The amount remaining in the credit card after 85 minutes is given by evaluating the above equation in x = 85.

y = -0.15*85 + 40 = 27.25

This means that after 85 minutes of calls, there are $27.25 left on the card.

If you want to learn more about linear equations:

https://brainly.com/question/1884491

#SPJ1

Statement: "Three less than four times a number is greater
than or equal to 41."

Answers

Answer:

x≥11

Step-by-step explanation:

4x-3≥41

4x≥41+3   -44

x≥44/4 = 11

A geometric seqerence 1.5, -3, 6, -12 .... How many regative terms in the sequencs greater than
- 6000 ?​

Answers

The total negative terms are 4 which are greater than -6000 if the geometric sequence 1.5, -3, 6, -12 ... with a common ratio -3.

What is a sequence?

It is defined as the systematic way of representing the data that follows a certain rule of arithmetic.

We have:

A geometric sequence 1.5, -3, 6, -12 ....

The common ratio r = -3/1.5 = -2

1.5, -3.6, 6, -12, 36, -108, 324, -972, 2916, -8748

The total negative terms = 4

Thus, the total negative terms are 4 which are greater than -6000 if the geometric sequence 1.5, -3, 6, -12 ... with a common ratio -3.

Learn more about the sequence here:

brainly.com/question/21961097

#SPj1

The law of cosines is a² + b² - 2abcosC = c². Find the value of 2abcosC.
A. 20
B. 40
C. 37
D. -40​

Answers

Answer:

C

Step-by-step explanation:

a = 4

b = 5

c = 2

C = arccos((a² + b² - c²) / 2ab)

C = arccos((16 + 25 - 4) / 2(4)(5))

C = arccos(37 / 40)

C = 22.33°

2abcosC

2(4)(5)cos(22.33)

40(0.925)

37

Option C is correct, if the law of cosines is a² + b² - 2abcosC = c² then the value of 2abcosC is 37.

What is Trigonometry?

Trigonometry is a branch of mathematics that studies relationships between side lengths and angles of triangles.

We have from law of cosines that a² + b² - 2abcosC = c²

We have to find the value of 2abcosC.

Now let us find the value of C

C = arccos((a² + b² - c²) / 2ab)

C = arccos((16 + 25 - 4) / 2(4)(5))

C = arccos(37 / 40)

C = 22.33°

Now plug in the value of C in  2abcosC.

2abcosC

=2(4)(5)cos(22.33)

=40(0.925)

=37

Hence, option C is correct, if the law of cosines is a² + b² - 2abcosC = c² then the value of 2abcosC is 37.

To learn more on trigonometry click:

https://brainly.com/question/25122835

#SPJ5

Please help, will give the brainliest!


(J) 80 • 125

(K) (80+65) • 125

(L) (65 • 80) + (80 • 60)

M (65•80)+(80•60)

N (65•80)+ 1/2 (125 • 100)

Answers

Answer:

option N is the correct option, apply Formula of area of rectangle and triangle by splitting figure into 2 parts

Value of x or y so that the line through the points has the given slope:

(X,9) and (9,-3), slope is -6/7

Sorry if it’s not worded great.

Answers

If we want the slope to be -6/7, then the value of x must be -5.

How to find the value of x?

Remember that if a linear equation passes through (x₁, y₁) and (x₂, y₂), then the slope is:

[tex]a = \frac{y_2 - y_1}{x_2 - x_1}[/tex]

Here we have the points (x, 9) and (9, -3), and the slope must be -6/7, so we need to solve:

[tex]\frac{- 3 - 9}{9 - x} = \frac{-6}{7} \\\\\frac{-12}{9 - x} = \frac{-12}{14}[/tex]

Where on the second line we multiplied and divided the right fraction by 2.

Comparing the fractions, we see that we must have:

9 -x = 14

9 - 14 = x

-5 = x

So the value of x must be -5.

If you want to learn more about linear equations:

https://brainly.com/question/1884491

#SPJ1

If a 103 pound man can have 1292 mg of a medicine in a day, how much can a 102 pound woman have? (round answer to one decimal place) She can have mg

Answers

Answer:

1279.5mg

Step-by-step explanation:

103 pound man takes -1292mg of medicines

102 pound woman will take -xmg of medicine

cross multiply

102x1292=103x

131784=103x

x=131784/103

x=1279.5mg

mark as brainliest

Women will have 1279.5mg of medicine in a day.

How much can a 102-pound woman have?

103-pound man takes -1292mg of medicines.

102-pound woman will take x mg of the medicine.

By cross multiply

102x1292=103x

131784=103x

x=131784/103

x=1279.5mg

What is an example of a word problem?

Word problems normally include mathematical modeling questions, where records and information approximately a positive machine is given and a student is needed to expand a model. For example, Jane had $5.00, then spent $2.00. How much does she have now?

Learn more about Word problems here: brainly.com/question/13818690

#SPJ2

The account balance of each of three children at the end of a month is shown below:


Daniel has −$1.25

Libby has −$2.00


Sandra has as much savings in her account as the amount Daniel owes the bank.


Part A: On a blank paper, draw a number line from −4 to +4. Plot points on the number line to show the account balances of the three children. Label the points using the name of each child. Describe in detail how you created and labeled this number line, and what it looks like in detail. (5 points)


Part B: Determine whether Daniel or Libby owes the bank more. Use absolute values and inequalities to write statements that justify your conclusion. (5 points)

Answers

Answer:

Using the concept of number line, we find that Libby owes more than Daniel.

Step-by-step explanation:

Given that the account balance of Daniel is -$1.25 and of Libby is -$2.00. Sandra has as much savings in her account as the amount Daniel owes the bank. So, her account balance is +$1.25.

The numbers to be plotted on the number line are -2.00, -1.25 and 1.25.

The number line is in the attachment.

These numbers are related as: -2 < -1.25 < 1.25.

From the number line, all the names that come left to the origin owe some amount to the bank while all the names that come right to the origin have some money as savings in their account.

Clearly, Daniel and Libby owe the bank.

But since Libby comes before Daniel, Libby owes more than Daniel.

For more explanation, refer the following link:

https://brainly.com/question/4727909

#SPJ10

How many square feet of tile is needed for a room of 24 foot x 24 foot

Answers

Answer:576

Step-by-step explanation:

24 squared is 576

Simplify each expression and state whether each is a volume, an area, or neither.
a) n(4)2 x
b) lw+lw+wh+wh+lh+lh
c) 8y2(10y2)

Answers

a. 2n^4b, neither a volume nor an area

b. 2Iw + 2wh + 2Ih, It is an area

c. 80 y^4, neither a volume nor an area

How to simply the algebraic expressions

a. n(4) 2 x b

n * n * n* n* 2 * b

2n^4b

It is neither an area nor a volume

b.  lw+lw+wh+wh+lh+lh

Collect like terms

2Iw + 2wh + 2Ih

It is an area

c. 8y2(10y2)
8y^2 * 10 * y^2

80 y^4

It is neither a volume nor an area

Thus, the expressions are simplified as a. 2n^4b, b. 2Iw + 2wh + 2Ih, c. 80 y^4

Learn more about algebraic expressions here:

https://brainly.com/question/4344214

#SPJ1

Find the equation of the line parallel to y = 2x - 4 that runs through the point (-2, 4).
y = 1/2x + 4

y = 2x + 8

y = 2x + 4

y = -1/2x + 8

Answers

Answer:

y = 2x + 8

Explanation:

Equation: y = 2x - 4

Comparing it with slope intercept form "y = mx + b" where 'm' is slope and 'b' is y-intercept. This function has slope: 2 and y intercept of -4

Parallel lines has same slope.

Passes through (-2, 4)

[tex]\sf y - y_1 = m(x - x_1)[/tex]

[tex]\sf y - 4 = 2(x - (-2))[/tex]

[tex]\sf y - 4 = 2(x + 2)[/tex]

[tex]\sf y = 2x + 4 + 4[/tex]

[tex]\sf y = 2x + 8[/tex]

Answer: y = 2x + 8

Step-by-step explanation:

This is a fairly simple problem. I hope this helps!

So, to find a line parallel, it's simple, and it only requires that for the other line to have the same slope and a different y-intercept (or same "m" and different "b" in the equation y = mx + b).

To find a line parallel to a point is slightly harder, but don't worry, I'll teach you how.

To do this, we have to recognize the point. The point given is (-2, 4) where -2 is the x-value and 4 is the y-value. Remember, points always go by (x, y).

So, we set y to 4, and we set x to -2.

When testing out lines that are parallel to a point, you must always be careful that you end up with the same value. Such as 4 = 4 or 8 = 8, etcetera.

We get:

4 = 2 × (-2) + 8

Simplifying, we get:

4 = -4 + 8

Adding -4 to 8, we get 4:

4 = 4

Thus, y = 2x + 8 is parallel to the point (-2, 4)

distribute 5x(3x+7)
help asap

Answers

Answer:

The Answer is 50 or 50x

Answer: 15x^2+35x ✅

Step-by-step explanation:

Hii, do you need to distribute [tex]\boldsymbol{5x(3x+7)}[/tex]? No problem! (:

How to Distribute?

We can solve problems like this one by using what is known as the "distributive property". What it says is: If you have a term that comes before the parentheses, you multiply it times every single term in the parentheses. Let me show you what I mean...

[tex]\twoheadrightarrow\sf 5x(3x+7)[/tex]

[tex]\bullet[/tex] Multiply 5x tmes 3x and 7 - you will obtain

[tex]\twoheadrightarrow\sf 15x^2+35x[/tex] (if we multiply x times x we obtain x²)

Voila! There's our solution, cheers! (:

--

Hope that this helped! Best wishes.

[tex]\it Reach\quad far.\:Aim\quad high. \,Dream\quad big.[/tex]

[tex]\bigstar\underbrace[/tex]

--

Find the area of the shaded portion of the
figure. All angles are right angles.
Dimensions are in inches.
3
8
5

Answers

Answer:  22 square inches

======================================================

Explanation:

The white rectangle is 3 by 6 with area of 3*6 = 18 square inches.

The larger rectangle is 8 by 5 with area of 8*5 = 40 square inches.

The difference of these will get us the shaded area:

40 - 18 = 22 square inches

A water tank in the shape of an inverted circular cone has a base radius of 3 m and height of 7 m . If water is being pumped into the tank at a rate of 4.3 m^3 / min , find the rate at which the water level is rising when the water is 3.5 m deep. (Round your answer to three decimal places if required)

Answers

The water level increases by 0.608 meters per minute when the water is 3.5 m deep

How to determine the rate?

The given parameters are:

Radius, r = 3Height, h = 7Rate in, V' = 4.3m^3/min

The relationship between the radius and height is:

r/h = 3/7

Make r the subject

r = 3h/7

The volume of a cone is;

[tex]V = \frac 13\pi r^2h[/tex]

This gives

[tex]V = \frac 13\pi (\frac{3h}{7})^2h[/tex]

Expand

[tex]V = \frac{3h^3}{49}\pi[/tex]

Differentiate

[tex]V' = \frac{9h^2}{49}\pi h'[/tex]

Make h' the subject

[tex]h' = \frac{49}{9\pi h^2}V'[/tex]

When the water level is 3.5.

We have:

[tex]h' = \frac{49}{9\pi * 3.5^2}V'[/tex]

Also, we have:

V' = 4.3

So, the equation becomes

[tex]h' = \frac{49}{9\pi * 3.5^2} * 4.3[/tex]

Evaluate the products

[tex]h' = \frac{210.7}{346.36}[/tex]

Evaluate the quotient

h' = 0.608

Hence, the water level increases by 0.608 meters per minute

Read more about volumes at:

https://brainly.com/question/10373132

#SPJ1

The slope of the line below is 0.8. Write the equation of the line in point-slope
form, using the coordinates of the labeled point. Do not use parenthesis on
the y side. Coordinates (-2,-3)

Answers

The equation of the line is y = 0.8x - 1.4 if the  slope of the line is 0.8 and line passes through coordinates (-2,-3)

What is a straight line?

A straight line is a combination of endless points joined on both sides of the point.

The slope 'm' of any straight line is given by:

[tex]\rm m =\dfrac{y_2-y_1}{x_2-x_1}[/tex]

The slope m = 0.8

The line passing through coordinates (-2,-3)

y = mx + c

y = 0.8x + c

Plug the point in the equation.

-3 = 0.8(-2) + c

c = -1.4

y = 0.8x - 1.4

Thus, the equation of the line is y = 0.8x - 1.4 if the  slope of the line is 0.8 and line passes through coordinates (-2,-3)

Learn more about the straight line here:

brainly.com/question/3493733

#SPJ1

What is the slope of the line that passes through the points (6, 2) and (4, 10)? −4 −14 14 4

Answers

Answer:

-4

Step-by-step explanation:

The equation for slope is:

[tex]m=\frac{y_2-y_1}{x_2-x_1}[/tex]

m: Slope

(6, 2) = [tex](x_1, y_1)[/tex]

(4, 10) = [tex](x_2, y_2)[/tex]

Substitute these values into the equation:

[tex]m=\frac{10-2}{4-6}=\frac{8}{-2}=-4[/tex]

Therefore the slope is -4.

Learn more about slope here:
https://brainly.com/question/3493733

SOLVING

[tex]\Large\maltese\underline{\textsf{A. What is Asked}}[/tex]

What is the slope of the line that passes through the points? 2 points given

[tex]\Large\maltese\underline{\textsf{B. This problem has been solved!}}[/tex]

The formula used here, is [tex]\bf{\dfrac{y2-y1}{x2-x1}}[/tex].

[tex]\rule{300}{1.7}[/tex]

[tex]\bf{\dfrac{10-2}{4-6}}[/tex] | subtract

[tex]\bf{\dfrac{8}{-2}}[/tex] | evaluate

[tex]\bf{-4}[/tex]

[tex]\rule{300}{1.7}[/tex]

[tex]\bf{Result:}[/tex]

           [tex]\bf{=Slope\;-4}[/tex]

[tex]\boxed{\bf{aesthetic\not101}}[/tex]

If you could please answer this that would be awesome!

Please graph it!

Answers

The graph is attached with the answer, This is looking like a horizontal staircase

What is a Step Function ?

A step function is a piece wise function whose all pieces are horizontal line or a point.

It is given in the question to graph the interval

-2 ≤x≤2

for f(x) = |x+3|

-2    1

-1     2

0     3

1      4

2     5

The graph is attached with the answer.

This is looking like a horizontal staircase

To know more about Step Function

https://brainly.com/question/118594

#SPJ1


Find the domain and range of the function graphed below. Write your answers in interval notation.

Answers

Answer:

Domain: [tex][1, \infty)[/tex]Range: [tex](-\infty, -2][/tex]

Step-by-step explanation:

The domain is the set of x-values and the range is the set of y-values.

Help me and i'll help you earn more points if the question is right

Answers

-21x^3 - 8x^3 = -29x^3
-7/4x^(1/2) - 0 = -7/4x^(1/2)
-6x^(1/4)
-12x^(-1/2)
12x^(-1/4)
14x^4

Therefore, the correct option is the 2nd one.

At school B, 108 year 8 students study French.
This is 60% of the year group.
How many students do not study French in year 8 in school B?

Answers

Step-by-step explanation:

108 = 60%

1% = 60%/60 = 108/60 = 1.8

100% = 1%×100 = 1.8×100 = 180

how many do not study French ?

the remainder of the students. that is 40% (100% - 60%) or 72 students (40×1.8 or simply 180-108).

Answer:

72 students do not study French.

Step-by-step explanation:

60% = 108 students

1. 108 ÷ 60 = 1.8 ( this is to find 1% of the students)

2. 1.8(1%) × 40 = 72

To check the answer:

108 + 72 = 180

180 × 60% = 108

What are the rotations that will carry this equilateral triangle onto itself?
A
B
90° counterclockwise rotation about its center P
270° counterclockwise rotation about its center P
120° counterclockwise rotation about its center P
240° clockwise rotation about its center P
O225 clockwise rotation about its center P
O200 counterclockwise rotation about its center P
rights reserved

Answers

The rotations that will carry the equilateral triangle in discuss onto itself are;

90° counterclockwise rotation about its center P.270° counterclockwise rotation about its center P.

Which rotations will carry this equilateral triangle onto itself?

It follows from the task content that the rotation which produces the desired output in which case, the rotation maps exactly unto the equilateral triangle is required.

On this note, when the rotation is 90° and 270° about the center P in which case, the rotation can be clockwise or anticlockwise, the desired result is obtained.

Read more on rotations;

https://brainly.com/question/98217

#SPJ1

Answer: 240 and 120

Step-by-step explanation:

Pls help me with my math

Answers

Answer:

d.

Step-by-step explanation:

Left line is defined when x < 1 (x is less than 1). The point is not full and that means that x = 1 is not included.

Right line is defined when x is greater or equal to one x ≥ 1.

Options that have x < 1 and x ≥ 1 are b and d, so the answer is one of those.

Equations of the lines are in slope-intercept form y = mx + b, where m is slope and b is y-intercept.

Right line has steeper slope than left line, so the slope of right line will have bigger absolute value. That is the case with option d. (Left line has slope -1 and right one has slope -2, absolute value of right slope is bigger.)

You could also check with y-intercepts. Left line has y-intercept at y = 2 and left line is defined when x < 1. Only option d meets these conditions.

The table shows the heights of three different plants.

Answers

Answer:

7/4 meters

Step-by-step explanation:

the tallest plant is 19/8

since they have all the same denominaters we can just compare the top numbers

so

the smallestp lant is 5/8

19/8 - 5/8 = 14/8

7/4 is your final answer after simplifying

The Rainbow Motor Motel is a 50-room facility with a restaurant to accommodate its guests. The motel is open 365 days a year, even on leap years; as Lurena Boucha, the proprietor, closes down the facility every February 29, her birthday, for an all day celebration of four years of successful operations. On the average, 98% of the rooms are available for sale. For the year just ended, the actual number of rooms sold totaled 15,000 and the number of guests for the year totaled 20,000. The projected occupancy rate for the next year is 82% of the available rooms based on past experience. The food information is as follows:

Sales $150,000, Beg Inventory 8,000, Purchases $70,000, Ending Inventory, $ 13,000 Consumption by employees (free of charge) $3,000, Number of checks 64,139. Cost of sales ?

Answers

From the information given, the cost of sales is $65,000. See the explanation below.

What is the definition of cost of sales?

Add your initial inventory to the purchases made over the time period, remove that amount from your ending inventory, and that is your cost of sales.

Cost of Sales = (Bi + P) - Ei

Where Bi is Beginning Inventory

P is Purchases; and

Ei is Ending Inventory.

So the derivation of the solution above is?

Beginning Inventory = 8,000

(add)          Purchase = 70,000

Bi + P                          = 78,000

Ending Inventory  = 13,000

Cost of sales = 78,000 - 13,000

= $65,000

Learn more about cost of sales at;
https://brainly.com/question/24561653
#SPJ1

If I=$312.50 R=25% T=0.25 what is P

Answers

Step-by-step explanation:

please mark me as brainlest

Transportation officials tell us that 60% of drivers wear seat belts while driving. Find the probability that more than 562 drivers in a sample of 900 drivers wear seat belts.

Answers

The  probability that more than 562 drivers is 0.9328

What is probability?

It is a branch of mathematics that deals with the occurrence of a random event.

Given: p = 0.60

n = 900

Let X denote the number of drivers that wear a seat belt,

P(X > 562)

[tex]\mu[/tex] = n*p = 900(0.60) = 540

[tex]\sigma^{2}[/tex]= n*p*(1-p)

=900*0.60*0.40

=216

P(X >562) = Pr( Z > 1.4976)

                = 0.9328

Hence, the probability that more than 562 drivers is 0.9328

Learn more about this concept here:

https://brainly.com/question/13853712

#SPJ1

         


In the 30-60-90 triangle below, side s has a length of.
length of
30
90⁰
S
10
60"
A. 5.2: 5.2
B. 5,10,3
and side q has a

Answers

Answer: C, 5, 5√3

Step-by-step explanation:

The short leg of a 30-60-90 triangle equal half the hypotenuse, 10/2 = 5. The long leg equals √3 times the short leg, 5*√3 = 5√3.

Other Questions
ageInstructions: Deconstruct the following conditional statement into its hypothesis and conclusion.Conditional Statement: "If it is raining, then there are clouds in the sky."Hypothesis: if you see rainConclusion: then there are clouds in the skyCheck Assume that the markets can no longer be segmented. What would be the quantity demanded if price is 50 in a monopoly market? What function represents this graph?? Read this excerpt from chapter 2 of The Scarlet Letter.The door of the jail being flung open from within, there appeared, in the first place, like a black shadow emerging into sunshine, the grim and grisly presence of the town-beadle, with a sword by his side and his staff of office in his hand.Which best describes the purpose of the words black shadow, grim, grisly, and sword?to impart a sorrowful moodto impart a solemn moodto establish the early morning settingto establish the militant jailhouse setting How would you describe the graph of y< - lxl?A dashed line up V on the (0,0) with the shading on the inside of the V.A solid line up V on the (0,0) with shading outside the V.A dashed line down V on the (0,0) with the shading inside the V. Describe the goals of the Bill and Melinda Gates Foundation. (Site 1) Cellulose is a starchy carbohydrate.Which of these statements aboutcellulose is TRUE?A. Cellulose is rarely found on Earth.B. Cellulose is a type of fat.C. Cellulose is a complex carbohydrate.D. Cellulose dissolves easily in water. What are the domain and range of the function represented by the set ofordered pairs?{(-7, 1), (-3, 2), (0, -2), (5,5)} 1 peasant changed rabbits to hens and 2 rabbits were equal to 3 hens, each hen produced the eggs that were equal to of the total number of hens, the peasant sold the eggs, 9 eggs will cost as many pennies as each hen that produced the eggs and totally he got 72 pennies so how many hens and how many rabbits he had? Kaspar Corporation makes a commercial-grade cooking griddle. The following information is available for Kaspar Corporations anticipated annual volume of 30,000 units. Direct materials $17 per unit Direct labor $ 8 per unit Variable manufacturing overhead $11 per unit Total Fixed manufacturing overhead $300,000 Variable selling and administrative expenses per unit cost $ 4 Fixed selling and administrative expenses $150,000 (Total cost) The company uses a 40% markup percentage on total cost. Required: (a) Compute the total cost per unit. Choose the correct form of the imperfect.Los Sevillanos ________ (desfilar) en la calle. find the value of xd:6 Fossil fuels represent Group of answer choices a large amount of paleozoic and mesozoic biomass buried anaerobically millions of years ago. natural deposits of organic materials that formed long before life on Earth evolved. the remains of fossil animals that lived a few thousand years ago. sustainable sources of energy that are underused today. A medieval singer who would be most likely to adapt an epic poem would be aA. minstrelB. Gregorian monkC. troubadourD. dulcimer Cells are for tissues as organs are to identify two of the ideals that were described in one of Americas foundational documents .. Then explain whether or notor to what degreethose ideals have continued to guide the thoughts and actions of Americans, including American political leaders. five sentences of using either and five sentences of using neihter (c) Find the number of sets where all three cards are the same for exactly $0$ attributes. (d) Find the number of sets where all three cards are the same for exactly $1$ attribute. Does the capital city of your country stand in the same relation to the rest of the country as London does to Britain? Find the value of y.